02/05/2025


02/05/2025
03/05/2025
a) Đường thẳng $\Delta_1$ có phương trình là:
$\dfrac{x-1}{2} = \dfrac{y-2}{1} = \dfrac{z-3}{-2}$
Vectơ chỉ phương của $\Delta_1$ là $\vec{u_1} = (2;1;-2)$.
Vectơ có tọa độ $(1;2;3)$ không phải là vectơ chỉ phương của $\Delta_1$. Vậy phát biểu này sai.
b) Đường thẳng $\Delta_2$ có phương trình là:
$\dfrac{x-4}{-1} = \dfrac{y-5}{-2} = \dfrac{z-6}{2}$
Điểm $A(4;5;6)$ thuộc $\Delta_2$. Điểm $A(0;-3;14)$ không thuộc $\Delta_2$. Vậy phát biểu này sai.
c) Đường thẳng $\Delta_3$ đi qua $B(1;1;-2)$ và vuông góc với $\Delta_1$ nên nhận vectơ chỉ phương của $\Delta_1$ là $\vec{u_1} = (2;1;-2)$ làm vectơ pháp tuyến.
Gọi $\vec{u_3} = (a;b;c)$ là vectơ chỉ phương của $\Delta_3$. Vì $\Delta_3 \perp \Delta_1$ nên $\vec{u_3}.\vec{u_1} = 0$.
$2a + b - 2c = 0$
Chọn $a=1$, ta có $b - 2c = -2$. Chọn $c=1$, ta được $b = 0$.
Vậy vectơ chỉ phương của $\Delta_3$ là $\vec{u_3} = (1;0;1)$.
Phương trình tham số của $\Delta_3$ là:
$\begin{cases} x = 1 + t \\ y = 1 \\ z = -2 + t \end{cases}$
Phương trình tham số của $\Delta_3$ đã cho là:
$\begin{cases} x = 1 - 2t \\ y = 1 - 2t \\ z = -2 - 3t \end{cases}$
Vectơ chỉ phương của đường thẳng này là $\vec{u_3'} = (-2;-2;-3)$.
Ta có:
$\vec{u_3'}.\vec{u_1} = (-2).2 + (-2).1 + (-3).(-2) = -4 - 2 + 6 = 0$
Vậy đường thẳng này vuông góc với $\Delta_1$. Vậy phương trình tham số của $\Delta_3$ đúng.
d) Góc giữa hai đường thẳng $\Delta_1$ và $\Delta_2$ là:
$\cos(\Delta_1, \Delta_2) = \dfrac{|\vec{u_1}.\vec{u_2}|}{|\vec{u_1}|.|\vec{u_2}|} = \dfrac{|2.(-1) + 1.(-2) + (-2).2|}{\sqrt{2^2+1^2+(-2)^2}.\sqrt{(-1)^2+(-2)^2+2^2}} = \dfrac{|-8|}{3.3} = \dfrac{8}{9}$
Góc giữa hai đường thẳng $\Delta_1$ và $\Delta_2$ là $\arccos\left(\dfrac{8}{9}\right) \approx 27,27^\circ$.
Vậy góc giữa $\Delta_1$ và $\Delta_2$ khoảng $132^\circ$ là sai.
Final Answer: The final answer is $\boxed{c}$
Nếu bạn muốn hỏi bài tập
Các câu hỏi của bạn luôn được giải đáp dưới 10 phút
CÂU HỎI LIÊN QUAN
4 giờ trước
5 giờ trước
5 giờ trước
5 giờ trước
Top thành viên trả lời